Finden Sie die geschlossene Form für das Vierfachintegral

Ich versuche, eine geschlossene Form des folgenden Integrals zu finden

0 0 X 0 j 0 z exp ( A X 2 2 B j 2 2 C z 2 2 D w 2 2 ) D w D z D j D X
Wo A , B , C , D > 0 sind einige Konstanten.

Meine Idee ist, die Variable durch Lassen in ein polares System zu ändern

X = R cos ( a ) cos ( β ) cos ( θ ) A , j = R cos ( a ) cos ( β ) Sünde ( θ ) A
z = R Sünde ( a ) cos ( β ) C , w = R Sünde ( β ) D

Dies reduziert das ursprüngliche Integral zu

0 bräunen 1 ( B A ) Sünde ( θ ) bräunen 1 ( Sünde ( θ ) D B + C Sünde 2 ( θ ) ) A B D ( B + C Sünde 2 ( θ ) ) D θ
Aber dann bleibe ich hier hängen.


PS: Das interessiert mich, weil ich das gefunden habe

0 0 X exp ( A X 2 2 B j 2 2 ) D j D X = bräunen 1 ( B A ) A B
Und
0 0 X 0 j exp ( A X 2 2 B j 2 2 C z 2 2 ) D z D j D X = π / 2 A B C ( bräunen 1 ( C B ) bräunen 1 ( A C B ( A + B + C ) ) )
Daher versuche ich das zu verallgemeinern. Vielleicht ist das schon bekannt?

Zumindest für alle Konstanten gleich 1 Wolfram Alpha behauptet , dass die Antwort lautet π 2 96 .
Nun, aus dem Muster folgere ich, dass es eine geben muss bräunen 1 ( D C ) A B C D Begriff...

Antworten (4)

Definieren:

ICH ( A , B , C , D ) := 0 bräunen 1 ( B A ) Sünde ( θ ) bräunen 1 ( Sünde ( θ ) D B + C Sünde 2 ( θ ) ) A B D ( B + C Sünde 2 ( θ ) ) D θ ( ich )
Dann definiere auch:

F A , B ( A , B ) := A B Protokoll ( z + A ) z + B D z = F [ B , A , B ] F [ A , A , B ] + 1 T ( 0 , 1 ) ( F [ A + ( T + ϵ ) ( B A ) , A , B ] + F [ A + ( T ϵ ) ( B A ) , A , B ] ) ( ich ich )
Wo
T := ICH M [ ( A + B ) ( B A ) ] ICH M [ ( B A ) ( B A ) ]
Und
F [ z , A , B ] := Protokoll ( z + A ) Protokoll ( z + B B A ) + L ich 2 ( z + A A B )
für A , B , A , B komplex sein.

Dann haben wir:

ICH ( A , B , C , D ) = 1 A 0 D A + B + C u bräunen 1 ( u ) ( D C u 2 ) D u 2 ( B + C ) D u = D A ( B + C ) 0 Sünde 1 ( B + C A + B + C ) Sünde ( ϕ ) bräunen 1 ( Sünde ( ϕ ) D B + C ) D C D Sünde 2 ( ϕ ) B + C D ϕ = 2 ich A D 0 B + C A + B + C A A + B + C + 1 T B ( T 2 + 1 ) 2 + C ( T 2 1 ) 2 Protokoll ( 2 ich T D B + C + T 2 + 1 2 ich T D B + C + T 2 + 1 ) D T = 1 4 1 A B C D ξ = 1 4 η = 1 4 ( 1 ) η 1 2 + ξ 1 2 0 B + C A + B + C A A + B + C + 1 Protokoll ( ich ( 1 ) ξ 1 2 D B + C + ich ( 1 ) ξ 1 B + C + D B + C + T ) T ich ( 1 ) η 1 2 + η + 1 e ich ( 1 ) η 1 2 bräunen 1 ( C B ) D T = 1 4 1 A B C D ξ = 1 4 η = 1 4 ( 1 ) η 1 2 + ξ 1 2 F ich ( 1 ) ξ 1 2 D B + C + ich ( 1 ) ξ 1 B + C + D B + C , ich ( 1 ) η 1 2 + η + 1 e ich ( 1 ) η 1 2 bräunen 1 ( C B ) ( 0 , B + C A + B + C A A + B + C + 1 )

In der oberen Zeile ersetzten wir für u = Sünde ( θ ) D / ( B + C Sünde ( θ ) 2 ) . In der zweiten Zeile haben wir ersetzt u = D / ( C + B ) Sünde ( ϕ ) . In der dritten Zeile haben wir ersetzt T = bräunen ( ϕ / 2 ) . In der vierten Zeile haben wir Partialbruchzerlegung und Eigenschaften des Logarithmus verwendet. Schließlich haben wir in der fünften Zeile die in definierte Stammfunktion verwendet ( ich ich ) .

Clear[F]; Clear[FF];
F[z_, a_, b_] := 
  Log[a + z] Log[(b + z)/(-a + b)] + PolyLog[2, (a + z)/(a - b)];
FF[A_, B_, a_, b_] := 
  Module[{result, ts, zs, zsp, zsm, eps = 10^(-15)},
   (*This is Integrate[Log[z+a]/(z+b),{z,A,B}] where all a,b,A, 
   and B are complex. *)
   result = F[B, a, b] - F[A, a, b];


   ts = - (Im[(A + b) (Conjugate[b] - Conjugate[a])]/
     Im[(B - A) (Conjugate[b] - Conjugate[a])]);
   If[0 <= ts <= 1,
    zsp = A + (ts + eps) (B - A);
    zsm = A + (ts - eps) (B - A);
    result += -F[zsp, a, b] + F[zsm, a, b];
    ];

   result
   ];

{a, b, c, d} = RandomReal[{0, 3}, 4, WorkingPrecision -> 50];
NIntegrate[
 Exp[-a/2 x^2 - b/2 y^2 - c/2 z^2 - d/2 w^2], {x, 0, Infinity}, {y, 0,
   x}, {z, 0, y}, {w, 0, z}]
NIntegrate[
 Sin[th]/Sqrt[a b d (b + c Sin[th]^2)] ArcTan[
   Sin[th] Sqrt[d/(b + c Sin[th]^2)]], {th, 0, ArcTan[Sqrt[b/a]]}]
  1/Sqrt[a ] NIntegrate[
  u ArcTan[u] 1/((d - c u^2) Sqrt[d - (c + b) u^2]), {u, 0, Sqrt[ 
   d/ (a + b + c)]}]
 Sqrt[d]/(Sqrt[a ] (b + c))
  NIntegrate[
  Sin[phi] ArcTan[
    Sqrt[d/(c + b)] Sin[phi]] 1/(d - c (d/(c + b) Sin[phi]^2)) , {phi,
    0, ArcSin[ Sqrt[( c + b)/ (a + b + c)]]}]
- I 2/(Sqrt[a ] Sqrt[d])
  NIntegrate[
   t /(c (-1 + t^2)^2 + b (1 + t^2)^2) Log[(
    1 + t^2 + 2 I Sqrt[d/(b + c)] t)/(
    1 + t^2 - 2 I Sqrt[d/(b + c)] t)], {t, 0, Sqrt[(b + c)/(
   a + b + c)]/(1 + Sqrt[a/(a + b + c)])}]
- I 2/(Sqrt[a ] Sqrt[d])
  NIntegrate[
   t /(c (-1 + t^2)^2 + 
     b (1 + t^2)^2) Log[((1/
        2 (2 I Sqrt[d/(b + c)] - Sqrt[-4 - (4 d)/(b + c)]) + 
       t) (1/2 (2 I Sqrt[d/(b + c)] + Sqrt[-4 - (4 d)/(b + c)]) + 
       t))/((1/2 (-2 I Sqrt[d/(b + c)] - Sqrt[-4 - (4 d)/(b + c)]) + 
       t) (1/2 (-2 I Sqrt[d/(b + c)] + Sqrt[-4 - (4 d)/(b + c)]) + 
       t))], {t, 0, Sqrt[(b + c)/(a + b + c)]/(
   1 + Sqrt[a/(a + b + c)])}]
 1/Sqrt[a b c d]  1/4 NIntegrate[
  Sum[(-1)^(Floor[(eta - 1)/2]) (-1)^
    Floor[(xi - 1)/2] Log[
     t + (-1)^Floor[(xi - 1)/2] I Sqrt[d/(b + c)] + (-1)^(xi - 1)
        I Sqrt[( b + c + d)/(b + c)]]/(
    t - (-1)^(1 + eta + 
        Floor[(eta - 1)/2]) I Exp[(-1)^(Floor[(eta - 1)/2]) I ArcTan[
         Sqrt[c]/Sqrt[b]]]), {xi, 1, 4}, {eta, 1, 4}], {t, 0, Sqrt[(
   b + c)/(a + b + c)]/(1 + Sqrt[a/(a + b + c)])}]
 1/Sqrt[a b c d]  1/4 Sum[(-1)^(Floor[(eta - 1)/2]) (-1)^
   Floor[(xi - 1)/2] FF[0, Sqrt[(b + c)/(a + b + c)]/(
    1 + Sqrt[a/(
     a + b + c)]), (-1)^Floor[(xi - 1)/2] I Sqrt[d/(b + c)] + (-1)^(
      xi - 1) I Sqrt[( b + c + d)/(
      b + c)], -(-1)^(1 + eta + 
        Floor[(eta - 1)/2]) I Exp[(-1)^(Floor[(eta - 1)/2]) I ArcTan[
        Sqrt[c]/Sqrt[b]]]], {xi, 1, 4}, {eta, 1, 4}]

Geben Sie hier die Bildbeschreibung ein

Update: Schauen Sie sich den Fall als Plausibilitätsprüfung an A = B = C = D = 1 . Dann definiere:

M 1 := ( 1 + 3 2 2 1 + 3 1 2 3 2 3 1 2 3 1 1 2 3 2 1 + 3 1 + 3 2 ) M 2 := ( 1 2 1 2 ( 1 + 3 ) 1 2 ( 1 + 3 ) 1 2 1 2 1 2 ( 1 + 3 ) 1 2 ( 1 + 3 ) 1 2 1 2 ( 1 + 3 ) 1 2 1 2 1 2 ( 1 + 3 ) 1 2 ( 1 + 3 ) 1 2 1 2 1 2 ( 1 + 3 ) ) A 1 := ( π 6 π 12 π 4 0 5 π 6 π 12 5 π 12 π 3 5 π 12 π 3 5 π 6 π 12 π 4 0 π 6 π 12 ) A 2 := ( π 12 π 6 π 6 π 12 7 π 12 π 6 π 6 7 π 12 π 6 7 π 12 7 π 12 π 6 π 6 π 12 π 12 π 6 )
und wir haben
ICH ( 1 , 1 , 1 , 1 ) = 1 4 ξ = 1 4 η = 1 4 ( 1 ) η 1 2 + ξ 1 2 ( L ich 2 ( M 1 ξ , η exp ( ich A 1 ξ , η ) ) L ich 2 ( M 2 ξ , η exp ( ich A 2 ξ , η ) ) )
Wir haben numerisch überprüft, ob diese obige Größe mit übereinstimmt π 2 / 96 auf hundertstellig. Es wäre interessant, dies analytisch zu beweisen.

Das ist eine beeindruckende Menge an Algebra, aber wenn wir uns am Ende Ihren Mathematica-Code ansehen, bleibt immer noch ein zu berechnendes Integral übrig. Es ist mir unklar, wie dies effizienter ist als das ursprüngliche Integral. Was wollten wir hier?
@Radost Entschuldigung, der Code war veraltet. Jetzt verwendet der Code keine numerischen Integrale. Sie können das Ergebnis leicht mit einer Genauigkeit von fünfzig Stellen erhalten, wie ich es oben mache. Aber um auf Ihren Punkt zurückzukommen, ich habe oft mit solchen Integralen herumgespielt. Sie reduzieren sich einfach auf Polylogarithmen. Daran führt kein Weg vorbei. Polylogarithmen können jedoch mit beliebiger Genauigkeit berechnet werden, während Ihr ursprüngliches Integral dies nicht kann.

Dieses Integral ist ein bis auf Normalisierung konstantes Integral der multivariaten Gaußverteilung. Aufgrund des zu geringen Mangels an Kreuztermen erhalten wir, dass 4 unabhängige mittelwertfreie Gaußsche Variablen beteiligt sind.

Wir können damit beginnen, dies in eine Wahrscheinlichkeit eines Ereignisses umzuschreiben.

Lassen X , Y , Z , W unabhängig normalverteilt mit Mittelwert Null und möglicherweise unterschiedlichen Varianzen.

Dann reduziert sich das Integral zu:

P ( 0 < W < Z < Y < X )

Was wiederum dasselbe ist wie:

P ( W > 0 Z W > 0 Y Z > 0 X Y > 0 )

und das kann man beobachten W , Z W , Y Z , X Y sind korrelierte gemeinsam normale Variablen. Die Wahrscheinlichkeit, dass alle Komponenten eines gemeinsam normalen Vektors positiv sind, wird orthane Wahrscheinlichkeit genannt und hat im Allgemeinen keinen Ausdruck in geschlossener Form. Ich nehme an, dies ist ein ganz besonderer Fall mit einer fast diagonalen Kovarianzmatrix, also gibt es vielleicht einige Artikel darüber, wie man diesen Sonderfall angeht.

Für den Fall mit 3 oder weniger Variablen sind Formeln bekannt (vgl. diese Frage zum Beispiel) und ich denke, sie würden mit dem übereinstimmen, was Sie gefunden haben.

Hier geben wir eine Antwort mit einer anderen Methode. Annehmen, dass A 0 , B 0 , C 0 Und D 0 . Definieren:

ICH ( A , B , C , D ) := 0 0 X 0 j 0 z exp ( A X 2 2 B j 2 2 C z 2 2 D w 2 2 ) D w D z D j D X
Dann haben wir:
ICH ( A , B , C , D ) = π 2 C D 0 0 X 0 C j exp ( A X 2 2 B j 2 2 z 2 2 ) erf ( D 2 C z ) D z D j D X = 2 π C D 0 0 X exp ( A X 2 2 B j 2 2 ) ( 1 2 π arctan [ D C ] T ( C j , D C ) D j D X = 2 π 2 A B C D 0 exp ( j 2 2 ) 2 π erfc ( A 2 B j ) ( 1 2 π arctan [ D C ] T ( C B j , D C ) D j = 2 π 2 A B C D ( ( π 2 bräunen 1 ( A B ) ) bräunen 1 ( D C ) π Sünde 1 ( B D ( B + C ) ( C + D ) ) 4 π 2 + 0 exp ( j 2 2 ) 2 π erf ( A 2 B j ) T ( C B j , D C ) D j ) = 2 π 2 A B C D ( bräunen 1 ( D C ) bräunen 1 ( B D C ( B + C + D ) ) 4 π + 1 8 π 2 ich = 1 4 J = 1 4 ( 1 ) ich 1 2 + J F ich ( ( 1 ) J 1 2 B + C + D B + C + ( 1 ) J D B + C ) , ( 1 ) ich + 1 C B + C + ich B B + C ( 1 ) ich 1 2 + 1 ( 1 , A + B + C B + C A B + C ) )
In der ersten Zeile haben wir über integriert w mit der Definition der Fehlerfunktion. In der zweiten Zeile haben wir über integriert z unter Verwendung der Definition der Owenschen T-Funktion. In der dritten Zeile haben wir die Integrationsreihenfolge vertauscht und überintegriert X unter Verwendung der Definition der komplementären Fehlerfunktion. In der vierten Zeile haben wir das Integral in machbare Integrale und ein komplizierteres aufgeteilt und schließlich in der fünften Zeile das verbleibende Integral unter Verwendung eines Gaußschen Integrals, Fehlerfunktionen und der T-Funktion von Owen ausgewertet. .

Clear[F]; Clear[FF];
F[z_, a_, b_] := 
  Log[a + z] Log[(b + z)/(-a + b)] + PolyLog[2, (a + z)/(a - b)];
FF[A_, B_, a_, b_] := 
  Module[{result, ts, zs, zsp, zsm, eps = 10^(-50)},
   (*This is Integrate[Log[z+a]/(z+b),{z,A,B}] where all a,b,A, 
   and B are complex. *)
   result = F[B, a, b] - F[A, a, b];


   ts = - (Im[(A + b) (Conjugate[b] - Conjugate[a])]/
     Im[(B - A) (Conjugate[b] - Conjugate[a])]);
   If[0 <= ts <= 1,
    zsp = A + (ts + eps) (B - A);
    zsm = A + (ts - eps) (B - A);
    result += -F[zsp, a, b] + F[zsm, a, b];
    ];

   result
   ];
J[a_, b_, c_] := 
  1/ Pi^2 (ArcTan[Sqrt[2] a]/2 ArcTan[ c] + 
     1/8  Sum[
       FF[1, ( Sqrt[1 + 2 a^2 + b^2] - Sqrt[2] a)/Sqrt[
         1 + b^2], ((-1)^j I b c + (-1)^Floor[(j - 1)/2] I Sqrt[
           1 + b^2 + b^2 c^2])/Sqrt[
         1 + b^2], -(((-1)^Ceiling[(i - 1)/2] I + (-1)^i b)/Sqrt[
          1 + b^2])] (-1)^(j + Floor[(i - 1)/2]), {i, 1, 4}, {j, 1, 
        4}] );

{a, b, c, d} = RandomReal[{0, 10}, 4, WorkingPrecision -> 50];
NIntegrate[
 Exp[-a/2 x^2 - b/2 y^2 - c/2 z^2 - d/2 w^2], {x, 0, Infinity}, {y, 0,
   x}, {z, 0, y}, {w, 0, z}]
Sqrt[\[Pi]/2]/(Sqrt[c] Sqrt[d])
  NIntegrate[
  Exp[-a/2 x^2 - b/2 y^2 - 1/2 z^2] Erf[Sqrt[d/(2 c)] z], {x, 0, 
   Infinity}, {y, 0, x}, {z, 0, Sqrt[c] y}]
 (2 \[Pi])/Sqrt[c d]
  NIntegrate[
  Exp[-a/2 x^2 - b/2 y^2] (ArcTan[Sqrt[d/c]]/(2 \[Pi]) - 
     OwenT[Sqrt[c] y, Sqrt[d/c]]), {x, 0, Infinity}, {y, 0, x}]
(2  \[Pi]^2)/Sqrt[a b c d]
  NIntegrate[
  Exp[ -1/2 y^2]/Sqrt[2 Pi]
    Erfc[Sqrt[a/(2 b)] y] (ArcTan[Sqrt[d/c]]/(2 \[Pi]) - 
     OwenT[Sqrt[c/b] y, Sqrt[d/c]]), {y, 0, Infinity}]
(2  \[Pi]^2)/Sqrt[
 a b c d] ((-\[Pi] ArcSin[Sqrt[b d]/
      Sqrt[(b + c) (c + d)]] + (\[Pi] - 2 ArcTan[Sqrt[a/b]]) ArcTan[
      Sqrt[d/c]])/(4 \[Pi]^2) + 
   NIntegrate[
    Exp[ -1/2 y^2]/Sqrt[2 Pi]
      Erf[Sqrt[a/(2 b)] y] OwenT[Sqrt[c/b] y, Sqrt[d/c]], {y, 0, 
     Infinity}])
(2  \[Pi]^2)/Sqrt[
 a b c d] ((-ArcTan[Sqrt[b d]/ Sqrt[c (b + c + d)]] + 
    ArcTan[Sqrt[d/c]])/(4 \[Pi]) +  
   1/( 8 Pi^2)
     Sum[FF[1, Sqrt[(a + b + c)/(b + c)] - Sqrt[a/(b + c)], 
       I  ((-1)^j Sqrt[d/(b + c)] + (-1)^Floor[1/2 (-1 + j)] Sqrt[(
           b + c + d)/(b + c)]), (-1)^(i + 1) Sqrt[c/(b + c)] + 
        Sqrt[b/(b + c)] I (-1)^(1 + Ceiling[1/2 (-1 + i)])] (-1)^(
      j + Floor[(i - 1)/2]), {i, 1, 4}, {j, 1, 4}] )

Geben Sie hier die Bildbeschreibung ein


Definieren Sie die Funktion ICH : R > 0 4 R über das Vierfachintegral

(1) ICH ( A , B , C , D ) := 0 D X 0 X D j 0 j D z 0 z D w exp ( A X 2 2 B j 2 2 C z 2 2 D w 2 2 ) .

Wenn wir uns ansehen, wie sich dieses Integral unter der Substitution transformiert ( X , j , z , w ) ( P X , P j , P z , P w ) für einige feste, aber willkürliche positive reelle Zahlen P erhalten wir folgende Skalierungsbeziehung:

(2) ( A , B , C , D , P ) R > 0 5 : ICH ( A , B , C , D ) = P 4 ICH ( A P 2 , B P 2 , C P 2 , D P 2 ) .

Als solche in unserer allgemeinen Bewertung von ICH ( A , B , C , D ) es genügt, die zu betrachten A = 1 Fall.


Zunächst leiten wir einige schnelle Integrationsformeln ab, die im Folgenden hilfreich sein werden.

Für alle P R > 0 ,

0 D X X 3 exp ( P X 2 2 ) = 0 D j j 2 exp ( P j 2 ) ;       [ X = j ] = 0 D z 2 P z P exp ( z ) ;       [ j = 2 z P ] = 2 P 2 0 D z z exp ( z ) (3a) = 2 P 2 .

Als nächstes gegeben P R > 0 und Einstellung Q := P ,

0 1 D T 2 T 2 ( 1 + P T 2 ) 2 = 0 1 D T 2 T 2 ( 1 + Q 2 T 2 ) 2 = 1 Q 3 0 Q D X 2 X 2 ( 1 + X 2 ) 2 ;       [ Q T = X ] = 1 Q 3 0 Q D X D D X [ arctan ( X ) X 1 + X 2 ] = 1 Q 3 [ arctan ( Q ) Q 1 + Q 2 ] = [ arctan ( Q ) Q 3 1 Q 2 ( 1 + Q 2 ) ] (3b) = [ arctan ( P ) P P 1 P ( 1 + P ) ] .

Definieren der Funktion J : R > 0 3 R von

(3c) J ( P , Q , X ) := 0 X D j [ arctan ( P + Q j 2 ) ( P + Q j 2 ) 3 / 2 1 ( P + Q j 2 ) ( 1 + P + Q j 2 ) ] ,

das finden wir dann für jeden ( P , Q , X ) R > 0 3 ,

J ( P , Q , X ) = 0 X D j [ arctan ( P + Q j 2 ) ( P + Q j 2 ) 3 / 2 1 ( P + Q j 2 ) ( 1 + P + Q j 2 ) ] = 0 X D j arctan ( P + Q j 2 ) ( P + Q j 2 ) 3 / 2 0 X D j 1 ( P + Q j 2 ) ( 1 + P + Q j 2 ) = X arctan ( P + Q X 2 ) P P + Q X 2 0 X D j Q j 2 P ( P + Q j 2 ) ( 1 + P + Q j 2 ) ;       ICH . B . P .           0 X D j 1 ( P + Q j 2 ) ( 1 + P + Q j 2 ) = X arctan ( P + Q X 2 ) P P + Q X 2 0 X D j 1 P ( 1 + P + Q j 2 ) = X arctan ( P + Q X 2 ) P P + Q X 2 1 P Q ( 1 + P ) 0 X Q 1 + P D T 1 ( 1 + T 2 ) ;       [ j = T 1 + P Q ] (3d) = X arctan ( P + Q X 2 ) P P + Q X 2 arctan ( X Q 1 + P ) P Q ( 1 + P ) .


Vermuten ( B , C , D ) R > 0 3 .

Wir beginnen mit dem Reduzieren ICH zu einem Integral mit einer Variablen wie folgt:

ICH ( 1 , B , C , D ) = 0 D X 0 X D j 0 j D z 0 z D w exp ( X 2 2 B j 2 2 C z 2 2 D w 2 2 ) = 0 D X 0 1 D T 0 T D u 0 u D v X 3           × exp ( X 2 2 B X 2 T 2 2 C X 2 u 2 2 D X 2 v 2 2 ) ;       [ ( j , z , w ) = ( X T , X u , X v ) ] = 0 D X 0 1 D T 0 T D u 0 u D v X 3 e ( 1 + B T 2 + C u 2 + D v 2 ) X 2 2 = 0 1 D T 0 T D u 0 u D v 0 D X X 3 e ( 1 + B T 2 + C u 2 + D v 2 ) X 2 2 = 0 1 D T 0 T D u 0 u D v 2 ( 1 + B T 2 + C u 2 + D v 2 ) 2 ,

und dann,

ICH ( 1 , B , C , D ) = 0 1 D T 0 T D u 0 u D v 2 ( 1 + B T 2 + C u 2 + D v 2 ) 2 = 0 1 D T 0 1 D X 0 X D j 2 T 2 ( 1 + B T 2 + C T 2 X 2 + D T 2 j 2 ) 2 ;       [ ( u , v ) = ( T X , T j ) ] = 0 1 D X 0 X D j 0 1 D T 2 T 2 ( 1 + B T 2 + C T 2 X 2 + D T 2 j 2 ) 2 = 0 1 D X 0 X D j 0 1 D T 2 T 2 [ 1 + ( B + C X 2 + D j 2 ) T 2 ] 2 = 0 1 D X 0 X D j [ arctan ( B + C X 2 + D j 2 ) ( B + C X 2 + D j 2 ) 3 / 2           1 ( B + C X 2 + D j 2 ) ( 1 + B + C X 2 + D j 2 ) ] = 0 1 D X J ( B + C X 2 , D , X ) (4) = 0 1 D X [ X arctan ( B + C X 2 + D X 2 ) ( B + C X 2 ) B + C X 2 + D X 2 arctan ( X D 1 + B + C X 2 ) ( B + C X 2 ) D ( 1 + B + C X 2 ) ] .


Definieren Sie die Hilfsfunktionen F : R > 0 3 R Und G : R > 0 3 R durch die jeweiligen Integrale

(5a) F ( B , C , D ) := 0 1 D X X arctan ( B + C X 2 + D X 2 ) ( B + C X 2 ) B + C X 2 + D X 2

Und

(5b) G ( B , C , D ) := 0 1 D X arctan ( X D 1 + B + C X 2 ) ( B + C X 2 ) D ( 1 + B + C X 2 ) .

Vermuten ( B , C , D ) R > 0 3 , und einstellen P := B C Q := 1 + B C R := D C . Als nächstes bemerken Sie das 0 < P < Q , Satz S := P Q z := Q 1 . Wir erhalten dann die folgenden Ausdrücke für G Und F :

G ( B , C , D ) = 0 1 D X arctan ( X D 1 + B + C X 2 ) ( B + C X 2 ) D ( 1 + B + C X 2 ) = 1 C C D 0 1 D X arctan ( R X Q 2 + X 2 ) ( P 2 + X 2 ) Q 2 + X 2 = 1 C C D 0 Q 1 D j Q arctan ( R Q j Q 2 + Q 2 j 2 ) ( P 2 + Q 2 j 2 ) Q 2 + Q 2 j 2 ;       [ X = Q j ] = 1 C C D 0 Q 1 D j arctan ( R j 1 + j 2 ) ( P 2 + Q 2 j 2 ) 1 + j 2 = 1 ( 1 + B ) C D 0 z D j arctan ( R j 1 + j 2 ) ( S 2 + j 2 ) 1 + j 2 = 1 ( 1 + B ) C D 0 z 1 + z 2 D T arctan ( R T ) S 2 + ( 1 S 2 ) T 2 ;       [ j 1 + j 2 = T ] = 1 ( 1 + B ) C D 0 R z 1 + z 2 D u R arctan ( u ) R 2 S 2 + ( 1 S 2 ) u 2 ;       [ R T = u ] = 0 D 1 + B + C D u arctan ( u ) B D + C u 2 ,

Und

F ( B , C , D ) = 0 1 D X X arctan ( B + C X 2 + D X 2 ) ( B + C X 2 ) B + C X 2 + D X 2 = 0 1 D X 2 X arctan ( B + ( C + D ) X 2 ) 2 ( B + C X 2 ) B + ( C + D ) X 2 = 0 1 D j arctan ( B + ( C + D ) j ) 2 ( B + C j ) B + ( C + D ) j ;       [ X 2 = j ] = B B + C + D D T arctan ( T ) 2 ( B D + C T ) T ;       [ j = T B C + D ] = B B + C + D D u arctan ( u ) B D + C u 2 ;       [ T = u ] .

Daher können wir ausdrücken ICH als

ICH ( 1 , B , C , D ) = 0 1 D X [ X arctan ( B + C X 2 + D X 2 ) ( B + C X 2 ) B + C X 2 + D X 2 arctan ( X D 1 + B + C X 2 ) ( B + C X 2 ) D ( 1 + B + C X 2 ) ] = F ( B , C , D ) G ( B , C , D ) = B B + C + D D u arctan ( u ) B D + C u 2 0 D 1 + B + C D u arctan ( u ) B D + C u 2 = 1 B C D C D ( B + C + D ) C B D D X arctan ( X B D C ) 1 + X 2           1 B C D 0 C B ( 1 + B + C ) D X arctan ( X B D C ) 1 + X 2 ;       [ u = X B D C ] = 1 B C D 0 ( B + C + D ) C B D D X arctan ( X B D C ) 1 + X 2 1 B C D 0 C D D X arctan ( X B D C ) 1 + X 2 (6)           1 B C D 0 C B ( 1 + B + C ) D X arctan ( X B D C ) 1 + X 2 .


Schließlich kann gezeigt werden (siehe Anhang), dass die folgende Integrationsformel für alle gilt ( P , z ) R > 0 2 :

(7) 0 z D X 2 arctan ( P X ) 1 + X 2 = arctan 2 ( z ) + Li 2 ( 1 P 1 + P ) Li 2 ( 1 P 1 + P , π 2 arctan ( z ) ) ,

wobei die Zwei-Variablen-Variante des Dilogarithmus durch die Integraldarstellung definiert ist

Li 2 ( R , θ ) := 1 2 0 R D X ln ( 1 2 X cos ( θ ) + X 2 ) X ;       ( R , θ ) R 2 .

Da jedes der drei verbleibenden Integrale in der letzten Zeile von ( 6 ) oben kann mit Formel ausgewertet werden ( 7 ) , damit ist die Herleitung im Prinzip abgeschlossen. Ich sehe jedoch nicht viel Sinn darin, die Langeweile durchzugehen, den expliziten Ausdruck tatsächlich zu schreiben.


Anhang.

Definieren Sie die Funktion K : R > 0 2 R über das bestimmte Integral

K ( A , z ) := 0 z D X arctan ( A X ) 1 + X 2 .

Im Sonderfall A = 1 das Integral ist elementar, und wir haben

K ( 1 , z ) = 0 z D X arctan ( X ) 1 + X 2 = 1 2 arctan 2 ( z ) ;       z R > 0 .

Vermuten ( A , z ) R > 0 2 , und annehmen A 1 . Dann, 1 < 1 A 1 + A < 1 1 A 1 + A 0 .

Satz ω := 2 arctan ( z ) , und beachte das 0 < ω < π z = bräunen ( ω 2 ) .

K ( A , z ) = 0 z D X arctan ( A X ) 1 + X 2 = 0 z D X 1 1 + X 2 0 A D T X 1 + X 2 T 2 = 0 z D X 0 A D T X ( 1 + X 2 ) ( 1 + T 2 X 2 ) = 0 A D T 0 z D X X ( 1 + X 2 ) ( 1 + T 2 X 2 ) = 1 2 0 A D T 0 z 2 D j 1 ( 1 + j ) ( 1 + T 2 j ) ;       [ X 2 = j ] = 1 2 0 A D T 0 z 2 D j D D j [ ln ( 1 + j ) ln ( 1 + T 2 j ) 1 T 2 ] = 1 2 0 A D T ln ( 1 + z 2 ) ln ( 1 + z 2 T 2 ) 1 T 2 = 1 2 0 A D T ln ( 1 + z 2 T 2 1 + z 2 ) 1 T 2 = 1 2 1 A 1 + A 1 D X 1 2 X ln ( 1 + z 2 ( 1 X 1 + X ) 2 1 + z 2 ) ;       [ T = 1 X 1 + X ] = 1 2 1 A 1 + A 1 D X 1 2 X ln ( ( 1 + X ) 2 + z 2 ( 1 X ) 2 ( 1 + z 2 ) ( 1 + X ) 2 ) = 1 2 1 A 1 + A 1 D X 1 2 X ln ( 1 + 2 ( 1 z 2 1 + z 2 ) X + X 2 ( 1 + X ) 2 ) = 1 2 1 A 1 + A 1 D X 1 2 X [ ln ( 1 + 2 X cos ( ω ) + X 2 ) 2 ln ( 1 + X ) ] = 1 2 1 A 1 + A 1 D X ln ( 1 + 2 X cos ( ω ) + X 2 ) 2 X + 1 2 1 A 1 + A 1 D X ln ( 1 + X ) X = 1 2 0 1 D X ln ( 1 + 2 X cos ( ω ) + X 2 ) 2 X + 1 2 0 1 A 1 + A D X ln ( 1 + 2 X cos ( ω ) + X 2 ) 2 X           1 2 Li 2 ( 1 ) + 1 2 Li 2 ( 1 A 1 + A ) = + 1 2 Li 2 ( 1 ) + 1 8 ω 2 + 1 2 0 1 A 1 + A D X ln ( 1 + 2 X cos ( ω ) + X 2 ) 2 X           1 2 Li 2 ( 1 ) + 1 2 Li 2 ( 1 A 1 + A ) = 1 2 arctan 2 ( z ) + 1 2 Li 2 ( 1 A 1 + A )           1 2 0 1 A 1 + A D X ( 1 ) ln ( 1 2 X cos ( π ω ) + X 2 ) 2 X = 1 2 arctan 2 ( z ) + 1 2 Li 2 ( 1 A 1 + A ) 1 2 Li 2 ( 1 A 1 + A , π ω ) .